CLAT 2011 Question Paper

Download CLAT Past Year Papers with Full Solutions

We have compiled the CLAT Papers (2008 onwards) into one neat, beautifully formatted bundle for you to download, view offline or print. You can download it by clicking below

Download CLAT Question Papers & Solutions

COMMON LAW ADMISSION TEST (CLAT) 2011 Question Paper


Time: 2 hours Total Marks: 200

CLAT 2011 Question Paper
Section 1 – English and Reading Comprehension


The questions in this section are based on a single passage. The questions are to be answered on the basis of what is stated or implied in the passage. Please note that for some of the questions, more than one of the choices could conceivably answer the question. However, you are to choose the best answer; that is, the response that most accurately and completely answers the question.

PASSAGE FOR QUESTIONS 1 TO 10

In 1954, a Bombay economist named A.D.
Shroff began a Forum of Free Enterprise, whose ideas on economic development were somewhat at odds with those then influentially articulated by the Planning Commission of the Government of India. Shroff complained against the ‘indifference, if not discouragement’ with which the state treated entrepreneurs.
At the same time as Shroff, but independently of him, a journalist named Philip Spratt was writing a series of essays in favour of free enterprise. Spratt was a Cambridge communist who was sent by the party in 1920s to foment revolution in the sub-continent. Detected in the act, he spent many years in an Indian jail. The books he read in the prison, and his marriage to an Indian woman afterwards, inspired a steady move rightwards. By the 1950s, he was editing a pro-American weekly from Bangalore, called Mysindia. There he inveighed against the economic policies of the Government of India.
These, he said, treated the entrepreneur ‘as a criminal who has dared to use his brains independently of the state to create wealth and give employment’. The state’s chief planner, P.C. Mahalanobis, had surrounded himself with Western leftists and Soviet academicians, who reinforced his belief in ‘rigid control by the government over all activities’. The result, said Spratt, would be ‘the smothering of free enterprise, a famine of consumer goods, and the tying down of millions of workers to soul- deadening techniques.’
The voices of men like Spratt and Shroff were drowned in the chorus of popular support for a model of heavy industrialization funded and directed by the governments. The 1950s were certainly not propitious times for free marketers in India. But from time-to-time their ideas were revived. After the rupee was devalued in 1966, there were some moves towards freeing the trade regime, and hopes that the licensing system would also be liberalized. However, after Indira Gandhi split the Congress Party in 1969, her government took its ‘left turn’, nationalizing a fresh range of industries and returning to economic autarky.

1. Which of the following statements can most reasonably be inferred from the information available in the passage
a) P.C. Mahalanobis believed in empowering private entrepreneurs and promoting free market
b) Phillip Spratt preferred plans that would create economic conditions favourable for a forward march by the private enterprise
c) Restrictions on free markets enriched large Indian companies
d) Philip Spratt opposed the devaluation of rupee in 1966.

2. Which of the following statements is least likely to be inferred from the passage
a) Acceptance of A.D. Shroff’s plans in the official circles smothered free enterprise in India
b) The views of the Forum of Free Enterprise ran against the conception of development then prevalent among the policy makers
c) A.D. Shroff believed that state should actively support the private sector
d) Philip Spratt had been educated in Cambridge.

3.
Select the statement that best captures the central purpose of this passage
a) Highlight that even though there were advocates for free-market and private enterprise in the early years of independent India, they were crowded out by others who supported a dominant role for state over private enterprise
b) Explain the politics behind Indira Gandhi’s decision to nationalise the banks
c) Demonstrate with the help of statistics how the preference of policy makers for Soviet-style economic policies prevented India’s economic growth
d) Establish that devaluation of rupee in 1966 was vindicated by subsequent experience.

4. Philip Spratt came to India because he
a) Fell in love with an Indian woman
b) Wanted to protest against the economic policies of the Indian Government.
c) Was offered the editorship of Mysindia.
d) Had been instructed to work towards the goal of inciting a revolution in India.

5. The author avers that A.D Shroffs ideas were somewhat at odds with the views of Planning Commission because
a) A.D. Shroff was in favour of rigid governmental control over all economic activities
b) Shroff had opposed government’s decision to devalue Indian rupee
c) The hostility of the government to private entrepreneurs was complained against by A.D. Shroff
d) Shroff had been critical of the influence of Soviet academicians over India’s economic policy.

6. The ideological shift of Philip Spratt to the right was caused by
a) The demise of the Soviet Union
b) The start of the weekly called Mys¬india
c) The books that he encountered in the prison
d) The dissolution of his first marriage to his college friend.

7. Select the statement that could be most plausibly inferred from this passage
a) Philip Spratt and A.D. Shroff were members of the Forum for Free Enterprise
b) The first two Five-Year Plans emphasised on the importance of private enterprise as the spearhead of economic growth.
c) P.C. Mahalanobis had mooted the expulsion of foreign firms like Coca Cola and IBM from India
d) The hopes that the licensing regime would be liberalized after the devaluation of Indian rupee were belied in the aftermath of the split in the Congress Party.

8. The author alludes to nationalization of industries in 1969 in order to
a) Show the contradictions between A.D. Shroff’s economic views and the official economic policies of the Government of India
b) Exemplify the shift of the Indira Gandhi led government to the ‘left’
c) Demonstrate the ideological changes in the world-view of Philip Spratt
d) Highlight the negative political repercussions of the decision to devalue the Indian currency.

9. “Neither Philip Spratt nor A.D, Shroff ………..able to convince Mahalanobis.” Select the most appropriate phrase out of the four options for filling the blank space in the aforesaid sentence.
a) were
b) are
c) was
d) is.

10. The word ‘inveighed’ in this passage means
a) Praised
b) Recited
c) Proclaimed
d) Remonstrated.

PASSAGE FOR QUESTIONS 11 TO 20

In Mann Joseph’s debut novel Serious Men, the protagonist, Ayyan Mani, is a scheming Dalit-Buddhist who almost gets away with passing off his partially deaf son, Adi, as a prodigy, a genius who can recite the first 1,000 prime numbers. The garb of satire—where almost every character cuts a sorry figure-gives the author the licence to offer one of the most bleak and pessimistic portrayals of urban Dalits. Despite his savage portrayal of Dalit (and female) characters—or perhaps because of it-Serious Men has won critical appreciation from a cross-section of readers and critics.
At a time when a formidable body of Dalit literature – writing by Dalits about Dalit lives – has created a distinct space for itself, how and why is it that a novel such as Serious Men, with its gleefully skewed portrayal of an angry Dalit man, manages to win such accolades? In American literature—and particularly in the case of African-American authors and characters-these issues of representation have been debated for decades. But in India, the sustained refusal to address issues related to caste in everyday life—and the continued and unquestioned predominance of a Brahminical stranglehold over cultural production-have led us to a place where non-Dalit portrayal of Dalits in literature, cinema and art remains the norm.
The journey of modem Dalit literature has been a difficult one. But even though it has not necessarily enjoyed the support of numbers, we must engage with what Dalits are writing—not simply for reasons of authenticity, or as a concession to identity politics, but simply because of the aesthetic value of this body of writing, and for the insights it offers into the human condition. In a society that is still largely unwilling to recognise Dalits as equal, rights- bearing human beings, in a society that is inherently indifferent to the everyday violence against Dalits, in a society unwilling to share social and cultural resources equitably with Dalits unless mandated by law (as seen in the anti-reservation discourse), Dalit literature has the potential to humanise non-Dalits and sensitise them to a world into which they have no insight. But before we can understand what Dalit literature is seeking to accomplish, we need first to come to terms with the stranglehold of non-Dalit representations of Dalits.
Rohinton Mistry’s A Fine Balance, published 15 years ago, chronicles the travails of two Dalit characters-uncle Ishvar and nephew Om Prakash—who migrate to Bombay and yet cannot escape brutality. While the present of the novel is set at the time of the Emergency, Ishvar’s father Dukhi belongs to the era of the anti-colonial nationalist movement. During one of Dukhi’s visits to the town, he chances upon a meeting of the Indian National Congress, where speakers spread the “Mahatma’s message regarding the freedom struggle, the struggle for justice,” and wiping out “the disease of untouchability; ravaging us for centuries, denying dignity to our fellow human beings.”
Neither in the 1940s, where the novel’s past is set, nor in the Emergency period of the 1970s- when the minds and bodies Ishvar and Qmprakash, are savaged by the state-do we find any mention of a figure like B.R. Ambedkar or of Dalit movements. In his ‘nationalist’ understanding of modem Indian history, Mistry seems to have not veered too far from the road charted by predecessors like Mulk Raj Anand and Premchand. Sixty years after Premchand, Mistry’s literary imagination seems stuck in the empathy-realism mode, trapping Dalits in abjection. Mistry happily continues the broad stereotype of the Dalit as a passive sufferer, without consciousness of caste politics.

11. Which of the following is the closest description of the central argument of this passage?
a) Manu Joseph’s novel presents a scathing portrayal of Dalits
b) Contemporary American literature is very cautious on politically correct representation of minorities
c) The last two decades have witnessed the rise of a very vibrant Dalit literature
d) Portrayal of Dalits by non-Dalits merely as passive victims has been the dominant norm in Indian literature, cinema and art.

12. According to this passage, Premchand and Mulk Raj Anand
a) Presented a stereotyped version of Dalit characters in their writings
b) Excelled in writing satires on social inequality
c) Were politically opposed to the views of B.R. Ambedkar
d) Were closely involved with the leadership of the nationalist movement.

13. The writer refers to the ‘anti-reservation discourse’ in order to argue that
a) Dalit literature has had a very difficult journey since its origins
b) Manu Joseph is viscerally opposed to Dalits
c) Persons belonging to the upper castes are inherently indifferent to routine violence against Dalits
d) Indian society is not yet ready to equitably share, on its own, social, cultural and political space with Dalits.

14. Which of the following statements is least likely to be inferred from this passage?
a) The author of Serious Men has used the literary device of satire to present an unflattering picture of women characters
b) Issues of representation of minorities have been debated extensively in American literature.
c) The writer of this passage believes that engagement with Dalits is necessary only because such engagement affirms the importance of identity politics
d) The writer believes that Rohinton Mistry presented a stereotypical representation of Dalits character in his book.

15. According to the information available in the passage, the writer attributes the prevalence of representation of Dalits by non-Dalits in literature, art and media to
a) The nationalist understanding of Indian history
b) Marginalisation of B.R Ambedkar from nationalist movement
c) The anti-reservation discourse
d) Brahminical control over cultural production.

16. Which of the following is not among the reasons suggested by the writer for engaging with Dalit writing?
a) Dalit literature has the potential to sensitize non-Dalits about the experiences of the former
b) Dalit writing is more authentic than representation of Dalits by non-Dalits
c) Dalit literature does not have the support of numbers
d) The aesthetic value of Dalit writing.

17. Which of the following statement cannot be inferred from the passage?
a) Upper-castes have dominated the instruments of cultural production in Indian society
b) Indian society is unwilling to recognise Dalits as equal, rights- bearing human beings
c) Dalit writers have carved out a space for writings on Dalit experience and world view
d) The judiciary in India, in its opposition to reservation, has betrayed its unwillingness to acknowledge Dalits as equal bearer of rights.

18. The writer of this passage is critical of Rohinton Mistry’s A Fine Balance for the reason that
a) It is an example of a book on Dalit characters by a Non-Dalit
b) The book suggests that Dalits are nothing more than passive sufferers without any agency
c) The book ignores the everyday violence that Dalits have to confront with
d) It bares the passive literary style of the author, Rohinton Mistry.

19. Which of the following words would be the best substitute for the word ‘formidable’ in this passage?
(Question Modified)
a) Bright
b) impressive
c) devious
d) dim.

20. “It is not as if Dalit movements ………..not active during the periods that form A Fine Balance’s backdrop.” Select the most appropriate choice, to fill in the blank in the above sentence.
a) is
b) was
c) were
d) are.

PASSAGE FOR QUESTIONS 21 TO 30

In recent weeks, the writers William Dalrymple and Patrick French, among others, have come before a fusillade of criticism in India, much of it questioning not their facts, not their interpretations, but their foreignness.
“Who gets to write about India?” The Wall Street Journal asked on Wednesday in its own report on this Indian literary feuding. It is a complicated question, not least because to decide who gets to write about India, you would need to decide who gets to decide who gets to write about India. Rather than conjecturing some Committee for the Deciding of the Deciding of Who Gets to Write About India, it might be easier to let writers write what they please and readers read what they wish.
The accusations pouring forth from a section of the Indian commentariat are varied. Some criticism is of a genuine literary nature, fair game, customary, expected. But lately a good amount of the reproaching has been about identity.
In the case of Mr. Dalrymple, a Briton who lives in New Delhi, it is—in the critics’ view – that his writing is an act of re-colonization. In the case of Mr. French, it is that he belongs to a group of foreign writers who use business-class lounges and see some merit in capitalist and therefore do not know the real India, which only the commentariat member in question does.
What is most interesting about these appraisals is that their essential nature makes reading the book superfluous, as one of my Indian reviewers openly admitted. (His review was not about the book but about his refusal to read the book.) The book is not necessary in these cases, for the argument is about who can write about India, not what has been written.
For critics of this persuasion, India surely seems a lonely land. A country with a millennial history of Hindus, Christians, Jews, Muslims and Buddhists living peaceably together; a country of hundreds of dialects in which so many Indians are linguistic foreigners to each other, and happily, tolerantly so; a country that welcomes foreign seekers (of yoga poses, of spiritual wisdom, of ancestral roots) with open arms: a country where, outside the elite world of South.
Delhi and South Bombay, I have not heard an Indian ask whether outsiders have a right to write, think or exist on their soil.
But it is not just this deep-in-the-bones pluralism that challenges the who-gets-to-write- about-India contingent. It is also that at the very heart of India’s multifarious changes today is this glimmering idea: that Indians must be rewarded for what they do, not who they are.
Identities you never chose – caste, gender, birth order – are becoming less important determinants of fate. Your deeds – how hard you work, what risks you take – are becoming more important.
It is this idea, which I have found pulsating throughout the Indian layers, that leaves a certain portion of the intelligentsia out of sync with the surrounding country. As Mr. French has observed, there is a tendency in some of these writers to value social mobility only for themselves. When the new economy lifts up the huddled masses, then it becomes tawdry capitalism and rapacious imperialism and soulless globalization.
Fortunately for those without Indian passports, the nativists’ vision of India is under demographic siege. The young and the relentless are India’s future. They could not think more differently from these literatis.
They savour the freedom they are gaining to seek their own level in the society and to find their voice; and they tend to be delighted at the thought that some foreigners do the same in India and love their country as much as they do.

21. Which of the following statements is least likely to be inferred from the passage:
a) Younger generations of Indians are more tolerant of foreign scribes who write about their country
b) The writer believes that a section of Indian intelligentsia is very hostile to upward economic mobility
c) Mr. William Dalrymple has been accused of recolonising India through his writings
d) Most of the criticism that has been recently directed at Patrick French has emphasized mainly on the writer’s underwhelming literary style.

22. Which of the following would be the best substitute for the word, ‘fusillade’ in the passage?
a) Barrage
b) Breach
c) Temper
d) Row.

23. The writer uses the phrase, ‘who-gets-to- write-about-India contingent’ in this passage to refer to
a) Foreign writers who have written books on India
b) Critics who have attacked foreign writers writing on India for their mere foreignness
c) Elite residents of South Delhi and South Bombay
d) Cultural pluralists.

24. The writer believes that the most peculiar aspect of the criticisms that Patrick French and Willaim Dalrymple have received is that
a) Most such condemnation has emerged from elite Indians
b) Such critics are hostile to upward immobility
c) These censures are not centered on the books of such writers or their literary styles but are targeted at their identity instead
d) These critics ignore the plural ethos of India.

25. Which of the following statements can be inferred from the passage
a) Ascriptive identities like caste, tribe, etc. are becoming more and more important with the passage of time
b) Patrick French believes that the new market friendly economic policies followed for the last decades have resulted in the rise of tawdry capitalism and rapacious imperialism
c) The writer is of the opinion that a section of the intelligentsia is divorced from the views of their compatriots
d) While India has historically been very hospitable to a variety of religions, it has not been equally open to linguistic foreigners.

26. According to the information available in the passage, the writer is of the opinion that
a) Writers like Patrick French do not know the real India
b) Most of the condemnation heaped on Dalrymple, French and himself has been on expected lines
c) India’s reputation of pluralism is cosmetic at best, one that hides deep rooted hatred towards foreigners
d) The new generation of Indians have internalized/ the idea that people should be rewarded for what they do and not who they are.

27. The writer refers to the history of Hindus, Christians, Jews, Muslims and Buddhists living peaceably together in India for millions of years in order to
a) Show India’s openness to foreigners who have visited Indian in the quest for yoga.
b) Argue that India is a country of hundreds of dialects
c) Demonstrate the religiosity pervading in an average Indian
d) India’s deep-in the bones pluralism.

28. The writer argues that the nature of criticism he, Dalrymple and French have received for their books renders reading their books superfluous because
a) Such criticism has been limited to a very small minority of Indians
b) These writers are popular among Indian youth, even among those who have not read their books
c) The literary styles of these writers are not the sole focus of such criticism
d) Such criticism is less about what has been written in their books than about who can write on India.

29. According to the passage, the question ‘who gets to write about India’ is complicated because
a) India has been historically open to and tolerant of foreign writers and artists
b) This issue can be satisfactorily resolved only if we can decide who gets to decide who gets to write about India
c) Ascriptive identities are becoming more and more important in a globalised world
d) This would result in a shift of attention from what has been written to who has written.

30. “But with many outsiders’ India-related books recently hitting bookstores there, the sensitivity – flared into a bout of vigorous literary nativism, with equally vigorous counterpunches.” Select the most appropriate choice to fill in the blank in the above sentence
a) Has
b) Have
c) Was
d) did.

PASSAGE FOR QUESTIONS 31 TO 40

If religion and community are associated with global violence in the trends of many people, then so are global poverty and inequality. There has, in fact, been an increasing tendency in recent years to justify policies of poverty removal on the ground that this is the surest way to prevent political strife and turmoil. Basing public policy—international as well as domestic—on such an understanding has some evident attractions. Given the public anxiety about wars and disorders in the rich countries in the world, the indirect justification of poverty removal—not for its own sake but for the sake of peace and quiet in the world – provides an argument that appeals to self- interest for helping the needy. It presents an argument for allocating more resources on poverty removal because of its presumed political, rather than moral, relevance.
While the temptation to go in that direction is easy to understand, it is a perilous route to take even for a worthy cause. Part of the difficulty lies in-the possibility that if wrong, economic reductionism would not only impair our understanding of the world, but would also tend to undermine the declared rationale of the public commitment to remove poverty. This is a particularly serious concern, since poverty and massive inequality are terrible enough in themselves, and deserve priority even if there were no connection whatsoever with violence. Just as virtue is its own reward, poverty is at least its own penalty. This is not to deny that poverty and inequality can – and do – have far reaching consequences with conflict and strife, but these connections have to be examined and investigated with appropriate care and empirical scrutiny, rather than being casually invoked with unreasoned rapidity in support of a ‘good cause.”
Destitution can, of course, produce provocation for defying established laws and rules. But it need not give people the initiative, courage, and actual ability to do anything very violent. Destitution can be accompanied not only by economic debility, but also by political helplessness. A starving wretch can be too frail and too dejected to fight and battle, and even to protest and holler. It is thus not surprising that often enough intense and widespread suffering and misery have been accompanied by unusual peace and silence.
Indeed, many famines have occurred without there being much political rebellion or civil strife or intergroup warfare. For example, the famine years in the 1840s in Ireland were among the most peaceful, and there was little attempt by the hungry masses to intervene even as ship after ship sailed down the river Shannon with rich food. Looking elsewhere, my own childhood memories in Calcutta during the Bengal famine of 1943 include the sight of starving people dying in front of sweetshops with various layers of luscious food displayed behind the glass windows, without a single glass being broken, or law or order being disrupted.

31. Select the statement that can be most plausibly inferred from the aforesaid passage
a) A society plagued by recurrent famines can never witness political revolution
b) Religious discrimination inevitably leads to violence and strife
c) Destitution of the masses leads to peace and social stability
d) Famines and starvation do not necessarily result in political rebellion.

32. The author believes that it may not be advisable to emphasise on the connection between poverty and violence as
a) Emphasis on such connection appeals only to self-interest of persons
b) Linking poverty and violence undermines the moral character of anti-poverty measures
c) The absence of any essential connection between poverty and violence may then weaken the very rationale of anti-poverty policies
d) There is no necessary link between poverty and inequality.

33. Which of the following best captures the central argument of this passage?
a) Religion is inextricably linked with violence
b) Famines may not necessarily result in civil unrest
c) Global poverty and inequality are one of the fundamental causes of global violence and strife
d) Basing anti-poverty programmes on the need for avoidance of violence and strife is dotted with many pitfalls.

34. In the given passage, the word ‘perilous’ means
a) Scared
b) Costly
c) Futile
d) Dangerous.

35. The author refers to his own experience as a child during the Bengal famine of 1943 in order to
a) Illustrate how religiosity may instill passive acceptance of even the worst forms of starvation among people
b) Repudiate the argument that religious discrimination usually tends to inspire violent protests
c) Substantiate his assertion that it is not unusual to have the most intense suffering and misery coexist with complete peace
d) Demonstrate that people confronted with acute starvation are rendered too helpless to protest ever at all.

36. The word ‘destitution’ in this passage can be best substituted by
a) Dejection
b) Indigence
c) Default
d) Dereliction.

37.
Which of the following statement is least likely to be inferred from the passage?
a) History is replete with instance of famines that have occurred without there being much violent protest
b) Many writers and critics are increasingly advocating for stronger policies on poverty removal on the ground that this would help prevent political turmoil
c) The author believes that the links between poverty and violence must never be emphasized at all
d) Economic debility in turn inhibits political freedom.

38. The author asserts that basing anti¬poverty measures on the avowed connections between poverty and violence has certain apparent benefits because
a) Poverty is similar to religious exploitation in terms of the potential violent consequences.
b) It leads to allocation of more resources on anti-poverty policies
c) The widespread concern about war and violence provides a rationale for poverty-removal that appeals to the ‘self-interest’ of persons
d) Otherwise, there would not have been the tendency to justify anti-poverty policies on the ground that they prevent political turmoil.

39. ‘Economic reductionism’ is this passage means
a) Neglecting the economic connection between poverty and violence
b) Excessive accent on poverty and inequality
c) Emphasizing on the linkage between violence, poverty and economic equality
d) The view that every conflict is caused by underlying economic tensions.

40. “A sense of encroachment, degradation and humiliation can be even easier,— mobilize for rebellion and revolt.” Select the most appropriate word out of the four options for filling the blank space in the aforesaid sentence
a) for
b) as
c) into
d) to.

CLAT 2011 Question Paper
Section II – GENERAL KNOWLEDGE

41. Why was Arundhati Roy investigated for sedition?
a) For committing contempt of court
b) For saying that Kashmir is not an integral part of India
c) For sympathising with the Maoists
d) For condemning nuclear tests conducted by India.

42. Damon Galgut’s “In a Strange Room’ was recently in news for
a) Man Booker Prize shortlist
b) Winning the Pulitzer Prize
c) Winning the Orange Prize for fiction
d) None of the above.

43. Who was recently in the news when the Supreme Court of India rejected her plea for Euthanasia, but paved the way for legalization of passive euthanasia?
a) Aruna Shanbaug
b) Aruna Roy
c) Mary Roy
d) Medha Patkar.

44. Nagoya Protocol, signed by India on 30 October, 2010 was
a) an international treaty of bilateral investment between India and Japan
b) an international treaty to ensure that local produce are exploited only under license and for the common good of the mankind
c) an international treaty to ensure that the benefits of natural resources and their commercial derivatives are shared with local communities
d) None of the above.

45. Julian Assange, the founder of Wiki¬Leaks, has been encamped in the Embassy at which nations? (Modified)
a) Ecuadari
b) Sweden
c) U.S.A.
d) Denmark.

46. Which of the following are the five countries that have decided to bid for 2017 World Athletics Championships?
a) Qatar, USA, China, Sri Lanka and Brazil
b) Germany, Britain, Hungary, Qatar and Spain
c) Germany, Qatar, India, Spain and China
d) Germany, Britain, China, Qatar and Spain.

47. The recent Tunisian revolution is known as
a) Orange Revolution
b) Jasmine Revolution
c) Purple Revolution
d) Crescent Revolution.

48. ‘The Naive and the Sentimental Novelist’ is a 2010 publication of Harvard University Press of which of the following authors?
a) Orhan Pamuk
b) J.M. Coetzie
c) Partha Chatterjee
d) Ben Okri.

49. Who replaced Rajiv Shukla as the IPL Chairman and Commissioner from 2013 year’s edition of the IPL? (Modified)
a) N. Sriniswasan
b) Ranjib Biswal
c) Ratnakar Shetty
d) Shashank Manohar.

50. Which one of the following films was officially selected to compete in the Un Certain Regard (A Certain Glance) category at the 2013 Cannes Film Festival?(Modified)
a) Fruilvale Station
b) My Name is Khan
c) Wednesday
d) Dhobi Ghat.

51. Ram Kumudini Devi whose birth centenary is being celebrated in 2011, was the
a) First woman barrister of India
b) First woman Mayor of Hyderabad
c) First woman photographer in India
d) First woman doctor of India.

52. The Supreme Court in 2010 upheld an order of the Bombay High Court to lift a four-year-old ban imposed by the Maharashtra government on publication and circulation of a controversial book, authored by American scholar James Laine. Identify the book from the following.
a) Chatrapati Shivaji
b) Shivaji-The Warrior King
c) Shivaji – The Hindu King in Muslim India
d) None of the above.

53. In which case did the Nagpur Bench of the Bombay High Court on July 14, 2010 commute the death sentence of six accused to igorous life imprisonment?
a) Khairlanji case
b) Bhopal Gas Leak case
c) Bhagalpur case
d) Nithari case.

54. The Shunglu panel was constituted for which of the following issues?
a) Investigate the 2G Spectrum Scam
b) Suggest Civil Service Reforms
c) Probe the Commonwealth Games Scam
d) Suggest reforms on centre-state relationship.

55. Who was appointed as the Chairman of the National Innovation Council in August 2010?
a) Shashi Tharoor
b) Chetan Bhagat
c) Arindam Chaudhury
d) SamPitroda.

56. Name the Kenya-born political lobbyist who runs a firm called Vaishnavi Corporate Communications, and has recently been in news?
a) Barkha Dutt
b) Vir Sanghhvi
c) Vina Ramani
d) Niira Radia.

57. Irom Sharmila has been fasting for the last 10 years to protest against which of the following issues?
a) Rape cases against Indian Army in Manipur
b) Emergency in Manipur
c) Prevention of Terrorism Act, 2005
d) Application of the Armed Forces (Special Powers) Act, 1958 in Manipur.

58. Thein Sein is the newly-appointed President of which of the following nations?
a) Indonesia
b) Malaysia
c) Myanmar
d) Thailand.

59. Baglihar dam has been a matter of dispute between which nations?
a) Bangladesh and Myanmar
b) Myanmar and China
c) India and China
d) India and Pakistan.

60. Who is the author of the book “TINDERBOX – The Past and Future of Pakistan”?
a) Husain Haqqanl
b) Yasmeen Niaz Mohiuddin
c) Ishrat Husain
d) M.J. Akbar.

61. On 25th January, 2011, BJP leaders Sushma Swaraj and Arun Jaitley were prevented from entering Srinagar and unfurl the national flag. What was that Rath Yatra called?
a) Swabhimaan Yatra
b) Ekta Yatra
c) Mukti Yatra
d) Swaraj Yatra.

62. Who is chairing the Joint Parliamentary Committee (JPC) on the 2G Spectrum allocation issue?
a) Murli Manohar Joshi
b) A Raja
c) M. Thambi Durai
d) P.C. Chacko.

63. Saina Nehwal recently defeated Ji Hyun Sung of South Korea to win which of the following titles?
a) Swiss Open Grand Prix Gold Badminton
b) Commonwealth Games
c) Singapore Open
d) Hong Kong Open.

64. ‘Moner Manush’, the film to win the ‘Golden Peacock’ at the 41st International Film Festival of India was based on the life of which legendary 19th century folk singer and spiritual leader?
a) Kabir
b) Surjya Sen
c) Kabir Suman
d) Lalan Fakir.

65. Justice P.C. Phukan Commission of Inquiry was constituted to enquire into which of the following incidents?
a) Clashes between Nagas and Lepchas in Nagaland on 14th August, 2008
b) Clashes between Bodos and Muslims in Northern Assam’s Udalguri district on 14th August, 2008
c) Clashes between Assamese and Bengalis in Guwahati on 14 August, 2008
d) None of the above.

66. The first woman Secretary-General of SAARC is from which country?
a) Maldives
b) Bhutan
c) Sri Lanka
d) India.

67. Under whose premiership was the Women’s Reservations Bill (to secure quotas for women in Parliament and State legislative assemblies) first introduced in Parliament?
a) Rajiv Gandhi
b) H.D. Deve Gowda
c) Atal Bihari Vajpayee
d) Manmohan Singh.

68. Which Irish player scored the fastest Century in the history of World Cup Cricket?
a) Niaal ‘OA1 Lten
b) Kevin O’ Brien
c) Cusack
d) Ed Joyce.

69. Which of the following report brought out the 2G Spectrum Scam?
a) CBI report
b) WikiLeaks
c) CAG report
d) None of the above.

70. In February 2011, Gopa Sabharwal was appointed as the first Vice-Chancellor of which University of international stature?
a) Jawaharlal Nehru University
b) Azim Premji University
c) Visva-Bharati University
d) Nalanda International University.

Are You Interested in India's Best CLAT Coaching?

Delhi Law Academy conducts India's best CLAT Coaching. We have coached thousands of students to success in CLAT. With some of India's most qualified faculty and the best technology, DLA's Online CLAT Coaching is the right choice!

71. With which Hindutva association are Sadhvi Pragya Singh Thakur and Swami Aseemanand allegedly associated?
a) Vishwa Hindu Parishad
b) Shri Ram Sena
c) Abhinav Bharat
d) Arya Samaj.

72. With which of the following do you associate the name P.J. Thomas?
a) Central Vigilance Commission
b) Lokpal
c) Anti-corruption Ombudsman
d) Banking Ombudsman.

73. The 18th Commonwealth Law Conference was held in which city in 2013?(Modified)
a) Delhi
b) Bangalore
c) Kolkata
d) Cape Town South Africa.

74. Gustavo Santaolall who composed the music to the song “Stranger Lives” in the movie “Dhobi Ghat”, is from which of the following nations?
a) Chile
b) Peru
c) Argentina
d) Mexico.

75. Which one of the following was not awarded a portion of the contested land by the judgment of the Allahabad High Court in 2010 pertaining to the Ayodhya dispute?
a) Nirmohi Akhada
b) Surmi Central Board of Waqfs
c) Rashtriya Swayamsevak Sangh
d) Bhagwan Sri Ram Lala Virajman.

76. Sania Mirza claimed silver in the tennis mixed doubles category in the Asian Games in, Guanzhou in November 2010. Who was her partner?
a) Mahesh Bhupathi
b) Somdev Dewarman
c) Leander Paes
d) Vishnu Vardhan.

77. China objected to the Dalai Lama’s recent visit to Tawang in Arunachal Pradesh. What was he visiting in Tawang?
a) A Buddhist monastery
b) A memorial to Tibetans massacred by the Chinese army
c) The residence of Panchen Lama
d) None of the above.

78. Which one of the following tribes lives in the Niyamgiri Hills, which is at the heart of the controversy surrounding Vedanta Resources’ mining operations?
a) Manna Dhora
b) Dongria Kondh
c) Pardhan
d) Mai Pahariya.

79. Which prominent Barrister-politician, who was closely linked with the emergency proclamation of 1975, breathed his last in Kolkata on 6th November 2010?
a) Jyoti Basu
b) Siddhartha Shankar Ray.
c) Hiren Mukherjee
d) Indrajit Gupta.

80. Who is the author of the book “Great Soul: Mahatma Gandhi and his Struggle with India’, criticised for its content?
a) Nirad C. Chaudhury
b) Joseph Lelyveld
c) Khushwant Singh
d) Hermann Kallenbach.

81. Which Gharana of classical singing did Late Pandit Bhimsen Joshi belong to?
a) Dhrupad
b) Maihar
c) Kirana
d) Etawah.

82. 14th March, 2011 was the 80th Anniversary of the first Indian Sound Film (talkie). Which Movie was it?
a) Jahan Ara
b) Alam Ara
c) Noorjehan
d) None of the above.

83. Which internationally renowned musician collaborated with Rahul Shanna to release a music album titled ‘Namaste India’?
a) Richard Clayderman
b) Kenny G
c) Yanni
d) Ricky Martin.

84. “War on Terrorism or American Strategy for Global Dominance” is authored by which of the following authors?
a) Noam Chomsky
b) Demetrios Caraley
c) Lea Brilmayer
d) ManzoorAlam.

85. Mohammad Asif, Mohammad Amir and Salman Butt (Cricket players of Pakistan) have been banned for being found guilty of spot fixing. To which of the following institutions have they appealed?
a) Pakistan Cricket Board
b) Anti-Corruption Tribunal of the Asian Cricket Council
c) Court of Arbitration for Sport
d) Anti-Corruption Tribunal of the International Cricket Council.

86. Indian driver Karun Chandok was recently in the news for which of the following?
a) Being selected as a reserve driver by Team Force India for the 2011 Formula One season
b) Being selected as a reserve driver by Team Lotus for the 2011 Formula One season
c) Being selected as a reserve driver by Team Ferrari for the 2011 Formula One season
d) None of the above.

87. The United Nations Framework Convention on Climate Change meeting of 2013 November was held in which of the following places?(Modified)
a) Colorado
b) Canberra
c) Warshw
d) None of the above.

88. Which of the following pairings is incorrect?
a) Muammar Gaddafi-Syria
b) Fidel Castro-Cuba
c) Pol Pot- Cambodia
d) Hosni Mubarak-Egypt.

89. The Right of Children to Full and Compulsoiy Education Act, 2009 requires private schools to ensure that… per cent of their students come from weaker sections and disadvantaged groups?
a) 2
b) 15
c) 25
d) 40.

90. Srikrishna Committee, which recently submitted its report, was constituted for which of the following issues?
a) Mumbai bomb blast case
b) Malegaon blast case
c) Telengana issue
d) 2G Spectrum corruption issue.

CLAT 2011 Question Paper
SECTION III-MATHEMATICS ABILITY

91. Akbar will turn 50 when his son Jehangir turns 18. What will be Akbar’s age when it will be exactly 5 times that of Jehangir?
a) 36
b) 40
c) 44
d) 48.

92. Arun can climb a Coconut tree by 1.5 feet by each lift; however he slips 0.5 feet every time he makes the next lift. How many individual lifts he will have to reach the top of the Coconut tree of 18.5 feet?
a) 20
b) 19
c) 18
d) 17.

93. Jogen’s taxable income for 2010-11 is Rs. 5,00,000. The tax rates are (i) nil for first 1,50,000, (ii) 10% for 1,50,001- 3,00,000, and (iii) 20% for the remaining. His Tax liability is
a) Rs. 45,000
b) Rs. 50,000
c) Rs. 55,000
d) Rs. 60,000.

94. The ratio of two numbers is 4 : 5. But, if each number is increased by 20, the ratio becomes 6 : 7. The sum of such numbers is
a) 90
b) 95
c) 100
d) 60.

95. During the academic session 2009-10, in Banaras Hindu University, Varanasi, the number of students studying Arts, Law and Commerce was in the ratio of 5 : 6 :
If during the academic session 2010-11 the number of students studying Arts, Law and Commerce increased by 20%, 30% and 40% respectively, what will be new ratio?
a) 26 : 42 : 63
b) 36 : 44 : 73
c) 26 : 39 : 49
d) 30 : 39 : 49.

96. Seema sold a mobile phone at the cost of Rs. 1,950 at a loss of 25%. At what cost will she have to sell it to get a profit of 30%? (Question Replaced)
a) Rs. 3,300
b) Rs. 2,600
c) Rs. 2,535
d) Rs. 3,380.

97. A man walks from his house to the Railway Station to catch a train, which is running as per schedule. If he walks at 6 km/hr, he misses the train by 9 minutes. However, if he walks at 7 km/hr, he reaches the station 6 minutes before the departure of train. The distance of his home to the Railway Station is
a) 2 km
b) 1.5 km
c) 1.05 km
d) 1.25 km.

98. Difference between two numbers is 9 and difference between their squares is 981. Lowest of the two numbers is
a) 40
b) 50
c) 55
d) 59.

99. Ms. Jhulan Goswami scores 102 runs in the 18th innings of her career and thus increases her average by 5. After the 18th inning, her average is
a) 17
b) 21
c) 26
d) 28.

100. In a staff room of 25 teachers, 13 drink black coffee, 7 milk coffee, 9 drink both tea and either type of coffee, and everyone drinks either of the beverages. How many teachers drink only tea?
a) insufficient information
b) 5
c) 6
d) 9.

101. A box contains 90 discs which are numbered from 1 to 90. If one disc is drawn at random from the box, the probability that it bears a perfect square number is
a) 1/10
b) 1/11
c) 1/90
d) 1/9.

102. Two coins are tossed simultaneously. The probability of getting at the most one head is
a) 1/4
b) 1/2
c) 3/4
d) 1

103. A flag pole 18 m high casts a shadow 9.6 m long. What is the distance of the top of the pole from the far end of the shadow?
a) 20 metres
b) 20.04 metres
c) 20.4 metres
d) 24 metres.

104. The 10th term of the series: 5, 8,11,14, … is
a) 32
b) 35
c) 38
d) 185.

105. A bag contains 19 red balls, 37 blue balls and 27 green balls. If a ball is picked up from this bag at random, what is the probability of picking a blue ball?
a) 19/83
b) 37/87
c) 34/81
d) None of the above.

106. A cylindrical tennis ball container can contain maximum three balls stacked on one another. The top and bottom balls also touch the lid and the base of the container respectively. If the volume of a tennis ball is 240 cm, then what is the volume of the container?
a) 1080cm3
b) 840 cm3
c) 1440 cm3
d) 720 cm3.

107. The area of a square is 225 sq. cm. which is equal to the area of a rectangle. The length of the rectangle is 16 cm. more than the breadth of the rectangle. What is the respective ratio between the side of the square and the breadth of the rectangle?(Question Replaced)
a) 3 : 5
b) 5:3
c) 5:4
d) 4 : 5.

108. If (9/7)3 x (49/81)2*6 =(7/9)9, then the value of x is
a) 12
b) 9
c) 8
d) 6.

109. Francis has 18 eggs out of which 12 eggs were sold at 10% loss than the cost price. At what mark up should he sell the remaining eggs to cover his losses?
a) 5%
b) 10%
c) 15%
d) 20%.

110. If the length and height of a brick increases by 10% each respectively, and the breadth reduces by 20%, what is the percentage change in the volume of the brick? (Options Modified)
a) 2.8
b) 3.0
c) 3.2
d) 3.6.

CLAT 2011 Question Paper
SECTION IV-LOGICAL REASONING

Note: The following six questions comprise of one or more statements. Please answer the questions on the basis of the given statement(s). Please make the factual assumptions required by the question even if you believe the statement is false.

111. ‘Where there is smoke, there is fire.’
Which of the following statements, if true, would show that the above statement is false?
a) There is sometimes smoke where there is no fire
b) There is sometimes fire where there is no smoke
c) There is no fire where there is no smoke
d) None of the above.

112. ‘Where there is poverty, there are always thieves’.
Which of the following statements, if true, would show that the above statement is false?
a) America is a rich country and there are thieves in America
b) Bhutan is a poor country and there are no thieves in Bhutan
c) Bangladesh is a poor country and there are many thieves in Bangladesh
d) Nepal is a rich country and there are no thieves in Nepal.

113. Statement 1: Sugar is bad for people with diabetes.
Statement 2: Leela does not eat sugar.
Assuming that Statements 1 and 2 are true, which of the following statements follows?
a) Leela has diabetes
b) Sugar is bad for Leela
c) People with diabetes do not eat sugar
d) None of the above.

114. Statement 1: People who read fashion magazines do not like to read fiction.
Statement 2: Tenzin does not read fashion magazines.
Assuming that Statements 1 and 2 are true, which of the following conclusions might be said to follow?
a) Tenzin likes to read fiction
b) Tenzin may or may not like to read fiction
c) Tenzin does not like to read fiction
d) Tenzin does not like fashion magazines.

115. Suleiman: All Communists are atheists. Sheeba: That is not true.
Which of the following, if true, would make Sheeba’s reply the most convincing?
a) My uncle is an atheist but he is not a Communist
b) My uncle is a Communist but he is not an atheist
c) My uncle is a Communist and an atheist
d) My uncle is neither a Communist not an atheist.

116. Statement 1: All pingos are Byronic
Statement 2: Shalisto is byronic.
Statement 3: Therefore
Fill in the blank.
a) Shalisto is a pingo
b) Shalisto is not a pingo
c) Shalisto is not byronic
d) None of the above.

For the following eight questions, study the sequence of letters, numbers or words carefully to work out the pattern on which it is based, and therefore what the next item in the sequence must be. For example, the sequence ‘A, C, E,
G,……..has odd-numbered letters of the alphabet; therefore, the next item must be T, The meanings of the words are irrelevant.

117. A, B, D, G, K, P,
What is the last alphabet in this sequence?
a) Z
b) Y
c) V
d) X.

118. B, C, E, G, K, M, Q, S,….
What is the next alphabet in this sequence?
a) T
b) U
c) V
d) W.

119. Z, X, T, N,….
What is the next alphabet in this sequence?
a) E
b) F
c) G
d) H.

120. ‘Apple, Application, Approval, Apricot, April’
Which of the following best fits in the blank?
a) Arrogant
b) April
c) Appropriate
d) Apiary.

121. ‘Sunday, Monday, Wednesday, Saturday, Wednesday, Monday, Sunday, Which of the following best fits in the blank?
a) Sunday
b) Tuesday
c) Saturday
d) Thursday.

122. ‘387924,……, 3724, 423, 32, 2. Which number is missing? (Options Modified)
a) 42978
b) 42783
c) 42983
d) 42683.

123. ‘Gym, hymn, lynx, pygmy, rhythm’
Which of the following words does not belong to the above set?
a) Myrrh
b) Mythic
c) Flyby
d) Syzygy.

124. Aadvark, Eerie, iiwi, Oolong,
Which of the following words follows the pattern of this series?
a) Uvula
b) Uulium
c) Uranium
d) Uranus.

For the following nine questions, read the given argument or statement carefully, making any factual assumptions necessary. Then choose the best answer out of the four choices to the question asked. Note that not all the facts given will be relevant for determining the answer.

125. ‘China has a higher literacy rate than India. This is due to the greater efficiency of the Communist system. Efficiency is sorely lacking in India’s democratic system. Therefore, democracy is the biggest obstacle to India’s achieving 100% literacy’.
Which of the following, if true, would directly undermine the above argument?
a) Inefficiency is equally a problem in democracies and Communist countries
b) Communist systems do not respect human rights
c) Freedom is more important than literacy
d) China is slowly making the transition to democracy.

126. ‘In 399 BC a jury in Athens condemned Socrates to death for impiety and corrupting the morals of the youth. Socrates’ friends offered to help him escape, but Socrates refused. Socrates argued that the fact that he had lived in Athens for so many years meant that he had committed himself to obeying its laws. It would therefore be wrong for him to break those very laws he was implicitly committed to obeying.’
Which one of the following claims constitutes the most plausible challenge to Socrates’ argument?
a) Long residence only commits someone to obeying just laws and Socrates was convicted under an unjust law
b) Long residence by itself does not imply a commitment to obeying laws since one never made any explicit commitment
c) Obedience to the law is not always required
d) There is no point in escaping from prison since one will anyway be captured again.

127. ‘Soft drinks have been shown by scientists to be bad for the teeth. Therefore, the government would be justified in banning all soft drinks from the Indian market’.
Assuming that the factual claim in the above argument is true, what else needs to be assumed for the conclusion to follow?
a) The government is justified in banning anything that is bad for dental health
b) Soft drinks are also bad for gastric health
c) Dental hygiene is a matter of great concern
d) No further assumptions are necessary.

128. ‘In order to be eligible for election to the Lok Sabha, a person must be at least 25 years of age. Moreover, one must not be bankrupt. Therefore, Jatinder Singh, over 50 years of age and without any criminal convictions, cannot be the Speaker of the Lok Sabha since he has just filed for bankruptcy.’
Which of the following must be assumed for the conclusion to follow logically?
a) Anyone over 50 years of age is eligible to be Speaker of the Lok Sabha as long as he or she has no criminal convictions
b) People without criminal convictions cannot be elected to the Lok Sabha even if they are not bankrupt
c) Only those eligible for election to the Lok Sabha are eligible to be the Speaker of the Lok Sabha
d) There is no minimum age requirement for the Speaker of the Lok Sabha.

129. ‘As a century draws to a close, people start behaving much like people coming to the end of a long life. People approaching death often start reflecting on the events of their lives. Similarly, people alive in 1999 ‘
Which of the following most logically completes the paragraph above?
a) .. .started reflecting on the events of the twentieth century
b) ….started to reflect on the events of their lives
c) … started to fear death
d) …started to wonder what the year 2000 would bring.

Are You Interested in India's Best CLAT Coaching?

Delhi Law Academy conducts India's best CLAT Coaching. We have coached thousands of students to success in CLAT. With some of India's most qualified faculty and the best technology, DLA's Online CLAT Coaching is the right choice!

130. ‘The Roman poet and philosopher Lucretius proposed the following thought experiment. If the universe has a boundary, we can throw a spear at this boundary. If the spear flies through, then it is not a boundary. If the spear bounces back, there must something beyond this boundary that is itself in space, which means it is not a boundary at all. Either way, it turns out that the universe has no boundary.’
How best can the form of Lucretius’ argument for the infinity of space be described?
a) Lucretius shows that positing finite space leads to a contradiction
b) Lucretius shows that finite space is inconsistent with physics
c) Lucretius shows that finite space is inconsistent with mathematics
d) Lucretius shows that finite space is inconceivable.

131. ‘Utilitarians believe that the right action is that which produces the most happiness.’
Which of the following claims is incompatible with the utilitarian view?
a) The right thing to do is to make the consequences of our actions as good as possible
b) The right thing to do is to do our duty, whatever the consequences
c) The right thing to do is to act on a rule which, if followed widely, produces the most happiness
d) The right thing to do is to act from motivations which produce the most happiness.

132. ‘Senthil goes to Ambala for the first time in his life. On the way from the railway station to his hotel, he sees twelve people, all of them male. He concludes that there are no women in Ambala. As a matter of fact, there are many thousands of women in Ambala.’
Which of the following best describes Senthil’s error?
a) Senthil was misled by irrelevant details
b) Senthil generalised on the basis of insufficient evidence
c) Senthil was biased against women
d) Senthil was bad at counting.

133. ‘It took many centuries before the countries of Europe could resolve their internal problems of violence and corruption to become the stable nation¬states they are. Therefore, it will take many centuries for India to achieve internal stability/
Assuming that the factual claims in the argument above are ture, what must be assumed in order for the conclusion to follow?
a) Countries everywhere must follow the same paths towards stability
b) India is gradually progressing towards internal stability
c) India can learn how to achieve stability by studying Europe’s example
d) India has no hope of achieving stability in the near future.

In the following three questions, assume that a ‘fact’ expresses something that can be proved by clear and objective data. An opinion expresses a judgment, view, attitude, or conclusion that is not backed by data.

134. Which one of the following statements is best described as an assertion of opinion rather than an assertion of fact?
Brazil, China and India are now among the largest emitters of greenhouse gases
a) Scientists agree that human activity is an important cause of climate change
b) The Indian Government’s policy on climate change is misguided
c) The Indian Government’s policy on climate change has changed significantly in the last five years.

135. ‘(A) The number of people migrating into Bengaluru has increased significantly in recent years. (B) This is because Bengaluru provides more economic opportunities than the towns and villages from which these migrants come. (C) This sudden influx of migrants has made the city less pleasant to live in. (D) The success of the government’s rural employment guarantee act might have the effect of stemming some rural-urban migration.’
Which one of the above statements is best described as an assertion of opinion rather than an assertion of fact?
a) A
b) B
c) C
d) D.

136. Which one of the following statements, if true, is best described as an assertion of opinion rather than an assertion of fact?
a) Mumbai is larger than Pune
b) Mumbai is smaller than Pune
c) Mumbai is more cultured than Pune
d) Mumbai is more crowded than Pune.

For the following three questions, there are two or more statements along with few conclusions deduced from the statements. You are required to answer on the basis of the statements and the conclusions. Make the factual assumptions required by the question even if you believe the statement is actually false.

137. Statement 1: Some aeroplanes are balloons.
Statement 2 : Some balloons are rockets. Conclusions:
a) Some aeroplanes are rockets
b) Some rockets are tables
c) All the rockets are balloons
d) All the balloons are aeroplanes.

Assuming that statements 1 and 2 are true, which conclusions follow:
a) Only (B) and (D)
b) Only (A) and (C)
c) Only (D)
d) None of the above.

138. Statement 1: All whales are fish.
Statement 2: Some fish are not amphibians.
Statement 3: All whales are amphibians.
Statement 4: Some amphibians are not fish.
Conclusions:
a) Some fish are amphibians
b) Some amphibians are fish
c) Only whales are both fish and amphibians
d) All amphibians are fish.

Assuming only that Statements 1,2,3 and 4 are true, which of the above conclusions may be deduced?
a) Only (A) and (B)
b) Only (C)
c) Only (D)
d) None of the above.

139. Statement 1: All libraries are laboratories.
Statement 2: No laboratories are hostels. Conclusions:
a) All laboratories are libraries
b) Some hostels are libraries
c) Some libraries are hostels
d) No library is a hostel.

Assuming that statements 1 and 2 are true, which conclusions follow?
a) Only (A) and (B)
b) Only (B) and (C)
c) Only (C) and (D)
d) Only D.

For the following sixteen questions, read the given passage carefully and answer the questions that follow.

PASSAGE FOR QUESTIONS 140-142

Instead of being concerned with what actually happens in practice … [economics] is increasingly reoccupied with developing pseudo-mathematical formulas. These provide models of behavour which never quite fit what actually happens, in a way which resembles the physical sciences; one wrong: instead of equations describing reality, economics produces equations describing deal conditions and theoretical clarity of a type which never .occurs in practice.

140. Which of the following best summarises the argument of this paragraph?
a) Economics ought to be more like the physical sciences
b) Theoretical clarity is undesirable in economics
c) The physical sciences are wrong to emphasise mathematic formulae
d) The mathematical equations used by economists do not accurately describe the real world.

141. Which of the following claims is not implied in the paragraph above?
a) Economists should stop using mathematical models
b) Equations describing ideal conditions should not be mistaken for equations describing reality
c) Theoretical clarity should not come at the expense of accuracy
d) Models of human behaviour should be true to the complexity of human nature.

142. Which of the following, if true, would most weaken the argument of the passage above?
a) The physical sciences are themselves reducing their reliance on mathematical formulae
b) The real world in fact closely approximates ideal theoretical conditions
c) We do not at present have the mathematical expertise to model the full complexity of the world economy
d) Academic economists need to be more sensitive to human nature.

PASSAGE FOR QUESTIONS 143-144

‘Religions, like camel caravans, seem to avoid mountain passes. Buddhism spread quickly south from Buddha’s birth-place in southern Nepal across the flat Gangetic plain to Sri Lanka. But it took a millennium to reach China … The religious belt stretched eventually to Mongolia and Japan, but in Afghanistan Buddhism filled only a narrow belt that left pagans among the valleys to the east and west in Kailash and Ghor’.

143. Which of the following best summarises the subject of this paragraph?
a) The Afghan people were hostile to Buddhism
b) Geography has considerable impact on the spread of religions
c) Buddhism does not flourish in mountainous regions
d) Religion has a considerable impact on geography.

144. Which of the following, if true, would most weaken the conclusion to the above argument?
a) Christianity took several centuries to cross the Atlantic Ocean to America
b) The Hindukush mountains made no difference to the speed with which Islam spread
c) Buddhism is strongest in mountainous regions
d) Jainism is less popular in cold climates.

PASSAGE FOR QUESTIONS 145-146

‘Friendship was indeed a value for the villagers, more for men than for women. Two good friends were said to be ‘like brothers’ (literally, ‘like elder brother-younger brother’, annatammandirahage). I heard this expression several times and I could not help recalling the statement of an elderly English colleague who had told me that he and his brother were very close and had written to each other every week. He had added, ‘We are very good friends.’ That is, friendship connoted intimacy in England while in Rampura (as in rural India everywhere), brotherhood conveyed intimacy’

145. Which of the following best summarises the conclusion of the argument of this paragraph?
a) Friendship has greater value for men than for women
b) People in England have different altitudes to brotherhood and friendship than people in rural India
c) Brotherhood has greater value in rural India than in England
d) Friendship has greater value in England than in India.

146. Which of the following, if true, would directly contradict the conclusions of the above argument?
a) People are less likely to have large families in England
b) People in England are no longer close to their family members
c) People in England do not think that friendship connotes intimacy
d) People in rural India think that sisters cannot be intimate.

PASSAGE FOR QUESTIONS 147-148

‘A language is most easily learnt when it is in tune with the social context. To teach an Indian child in English at the primary stage ……..strengthens distinctions of class and status and warps the mind. Failure to resort to regional languages in literacy campaigns also hampers their success’.

147. Which of the following best summarises the argument of the passage above?
a) Learning English warps the mind
b) Language policy should be devised with an eye to social context
c) Literacy campaigns in India have failed
d) English should be taught at the secondary school level.

148. Which of the following claims, if true, would weaken the argument in the passage above?
a) Literacy campaigns are more successful when conducted in English
b) Learning English at an early age has been found to weaken class distinctions
c) Children who learn English at the primary-school stage have been found to show greater intellectual ability than those who team English only at the secondary level
d) All of the above.

PASSAGE FOR QUESTIONS 149-151

‘The tribes should develop their own culture and make their contributions to the cultural richness of the country…it is unnecessary to cause them to change their customs, habits or diversions so far as to make themselves indistinguishable from other classes. To do so would be to rob rural and pastoral life of its colour and stimulating diversity’.

149. Which of the following conclusions is not implied by the passage above’?
a) It is good for India’s tribal people to develop their culture
b) Tribunal customs should not be allowed to change in any respect
c) Forcing tribal people to change their customs reduces the diversity of rural life
d) Tribal culture is part of the cultural richness of India.

150. Which of the following claims runs directly counter to the spirit of the passage above?
a) Tribal people should be able to decide what elements of the modem world to adopt
b) The government should make modem science and medicine available to tribal people
c) Tribal people should not be subjected to any coercion to conform to non- tribal cultural norms as long as they do not violate the law
d) The tribals should assimilate as far as possible into non-tribal culture as a conditions of full citizenship.

151. Which of the following is not an assumption required by the above argument?
a) Colour and diversity are desirable things
b) Tribal people are capable of contributing to India’s cultural diversity
c) Changing tribal customs is a necessary condition of making modem medicine available to tribal people
d) Rural life is presently full of colour and diversity.

PASSAGE FOR QUESTIONS 152-153

‘India is the only country in the world where, in the States which are governed by the Communist Party, human rights are fully respected — and that is only because the Bill of Rights is firmly entrenched in our national Constitution. We can proudly say that our Constitution gave us a flying start and equipped us adequately to meet the challenges of the future.

152. Which of the following conclusions is not implied by the passage above?
a) Communist states often do not respect human rights
b) Communist states never respect human rights
c) The entrenchment of the Bill of Rights is what ensures that even Communist- governed states respect human rights
d) The Indian Constitution prepared India for the challenges of independence.

153. Which of the following, if true, would weaken the above argument?
a) Communist governments are motivated to respect human rights out of Communist principles, not Constitutional ones
b) The Constitution of India is itself sympathetic to Communist principles
c) Human rights need to be enforceable to have any meaning
d) Countries without a strong culture of human rights are prone to oppressing minorities.

PASSAGE FOR QUESTIONS 154-155

‘Poverty is more restrictive and limiting than anything else. If poverty and low standards continue then democracy, for all its fine institutions and ideals, ceases to be a liberating force. It must therefore aim continuously at the eradication of poverty and its companion unemployment. In other words, political democracy is not enough. It must develop into economic democracy also.’

154. Which of the following is not implied by the above passage?
a) Democracy has ceased to be a liberating force
b) Democracies should aim to eliminate poverty
c) Poverty and unemployment go hand- in-hand
d) Political democracy should develop into economic democracy.

155. Which of the following views, if true, would weaken the argument of the above passage?
a) Political democracy is inseparable from economic democracy
b) Poverty does not in fact restrict freedom
c) Democracy flourishes most in poor societies
d) Economic democracy is a necessary condition for the elimination of unemployment.

CLAT 2011 Question Paper
SECTION V-LEGAL REASONING

Instructions to Candidates:
This section consists of ten problems (with 45 questions) in total. Each problem consists of a set of rules and facts. Apply the specified rules to the set of facts and answer the questions. In answering the following questions, you should not rely on any rule(s) except the rule(s) that are supplied for every problem. Further, you should not assume any fact other than those stated in the problem. The aim is to test your ability to properly apply a rule to a given set of facts, even when the result is absurd or unacceptable for any other reason. It is not the aim to test any knowledge of law you may already possess.

Problem 1 (For questions 156 to 160)
Rules
A. The fundamental right to freedom of association includes the right to form an association as well as not join an association.
B. The fundamental right to freedom of association also includes the freedom to decide with whom to associate.
C. C The fundamental right to freedom of association does not extend to the right to realise the objectives of forming the association.
D. Fundamental rights are applicable only to laws made by or administrative actions of the State and do not apply to actions of private persons.
E. Any law in contravention of fundamental rights is unconstitutional and therefore cannot bind any person.
Facts
Gajodhar Pharmaceuticals, a private company, offered an employment contract of two years to Syed Monirul Alam. One of the clauses in the employment contract provided that Syed Monirul Alam must join Gajodhar Mazdoor Sangh (GMS), one of the trade unions active in Gajodhar Pharmaceuticals.

156. Decide which of the following propositions can be most reasonably inferred through the application of the stated legal rules to the facts of this case
a) The employment contract offered to Monirul Alam to join GMS is legal as it does not restrict his freedom not to join any association
b) The condition requiring Monirul Alam to join GMS cannot bind him as it impinges on his freedom not to join any association
c) Syed Monirul Alam cannot claim a fundamental right to freedom of association against Gajodhar Pharmaceuticals and therefore, the contract would bind him even though his freedom of association is restricted
d) The employment contract infringes Syed Monirul Alam’s freedom to decide with whom to associate and therefore is legally not enforceable.

157. If Parliament enacts a law which requires every employee to join the largest trade union in their workplace mandating Syed Monirul Alam to join GMS, then
a) Such a law would merely govern private action to which fundamental rights do not apply
b) Such a law would not curtail any individual’s right to freedom of association
c) Neither the employment contract, nor the law of the Parliament would be enforceable as they would curtail the freedom of association
d) The law of Parliament would violate an individual’s freedom not to join any association and therefore be unconstitutional.

158. If Parliament enacts a law that requires a trade union to open its membership to all the employees, then
a) Such a law would not infringe any fundamental right to freedom of association
b) The law of the Parliament would curtail an individual’s right not to join any association
c) Such a law would curtail the union members’ right to decide with whom they would like to associate
d) Such a law would render the employment contract offered by Gajodhar Pharmaceuticals to Syed Monirful Alam unenforceable.

159. If Gajodhar Pharmaceuticals enter into an agreement with GMS wherein the former agrees to hire only the existing members of GMS as employees, then
a) The agreement would be illegal as it would curtail the union members’ right to decide with whom they would like to associate
b) Such an agreement would infringe the union’s right to decide with whom to associate and therefore is legally not enforceable
c) The agreement would not be enforceable as it would infringe upon the employer’s right not to join an association
d) The constitutionality of this agreement cannot be contested on grounds of contravention of fundamental rights as such rights are not applicable to private persons.

160. If Parliament enacts a legislation prohibiting strikes by trade unions of employees engaged in pharmaceutical industry, then
a) The legislation would not violate the right to freedom of association
b) The legislation would curtail the right of trade unions to strike, and therefore violate freedom of association
c) Since strike is only one of the objectives with which a trade union is formed, right to strike is not protected by the right to freedom of association
d) None of the above.

Problem 2 (For questions 161 to 163)

Rule: Whoever finds an unattended object can keep it unless the true owner claims that object. This does not affect the property owner’s right to the ownership of the property on which the object is found. The right to ownership of a property does not include the right to ownership of unattended objects on that property.
Facts: Elizabeth is the CEO of a global management services company in Chennai and is on her way to Ranchi to deliver the convocation address at India’s leading business school on the outskirts of Ranchi. Flying business class on Dolphin Airlines, she is entitled to use the lounge owned by the airline in Chennai Airport while waiting for her flight. She finds a diamond earring on the floor of the lounge and gives it to the staff of Dolphin Airlines expressly stating that in the event of nobody claiming the earring within six months, she would claim it back. The airline sells the earring after eight months and Elizabeth files a case to recover the value of the earring from the airline when she is informed about its sale.

161. As a judge you would order that
a) Elizabeth is not entitled to compensation because the earring was found on the property of the airline and therefore, the airline is entitled to sell it
b) The airline must compensate Elizabeth because owning the lounge does not give the airline the right over all things that might be found on it
c) The airline must compensate Elizabeth because while accepting the earring from Elizabeth they had agreed to return it if nobody claimed it within six months
d) Elizabeth is not entitled to compensation because she did not claim the earring after the expiry of six months and the airline waited for a couple more months before selling it.

162. Assume now that Elizabeth was only an economy class passenger and was not entitled to use the airline’s lounge. However, she manages to gain entry and finds the earring in the lounge. The rest of the above facts remain the same. Will her illegal entry into the lounge affect Elizabeth’s right to keep the earring (or be compensated for its value)?
a) Yes, the airline claims that Elizabeth’s entry into the lounge was illegal and therefore she has no right over anything she found there
b) No, because Elizabeth’s class of travel has no bearing on the outcome in this case
c) Cannot be determined as we need to know how Elizabeth was able to access the airline’s lounge
d) None of the above.

163. To the original fact scenario, the following fact is added: In the lounge there are numerous signboards which proclaim “Any unattended item will be confiscated by Dolphin Airlines’. In this case, you would
a) Order the airline to pay compensation to Elizabeth because the board in the lounge cannot grant property rights over unattended objects to the airline
b) Deny Elizabeth compensation because the signboard makes it evident that the airline, as owner of the lounge, is exercising all rights over all unattended items in the lounge and the earring is one such item
c) Deny Elizabeth compensation because she knew any unattended item belonged to the airline
d) Order the airline to pay compensation to Elizabeth because the property rights of the airline are relevant only if the item is unattended. The moment Elizabeth found the earring, it belonged to her.

Problem 3 (For Questions 164 to 168)

Rules A: The State shall not discriminate, either directly or indirectly, on the grounds of sex, race, religion, caste, creed, sexual orientation, marital status, disability, pregnancy, place of birth, gender orientation or any other status.
Rule B: Direct discrimination occurs when for a reason related to one or more prohibited grounds a person or group of persons is treated less favourably than another person or another group of persons in a comparable situation.
Rule C: Indirect discrimination occurs when a provision, criterion or practice which is neutral on the face of it would have the effect of putting persons having a status or a characteristic associated with one or more prohibited grounds at a particular disadvantage compared with other persons.
Rule D: Discrimination shall be justified when such discrimination is absolutely necessary in order to promote the well-being of disadvantaged groups, such as women, dalits, religious minorities, sexual minorities or disabled persons.
Facts: On 2nd October, 2010, the Governor of the State of Bihar ordered the release of all women prisoners who were serving sentence of less than one year imprisonment to mark the occasion of Mahatma Gandhi’s birthday.

164. Which of the following is correct with respect to the Governor’s order?
a) It discriminates directly on the ground of sex
b) It discriminates indirectly on the ground of sex
c) It does not discriminate on the ground of sex
d) It discriminates directly as well as indirectly on the ground of sex.

165. Is the governor’s order justified under Rule D?
a) Yes, because it is for the well-being of women prisoners.
b) No because it is not absolutely necessary for the well-being of women prisoners
c) No, because it does not promote the well-being of women prisoners or the society
d) None of the above.

166. Assume that the Governor also made a second order requiring the release of all persons under the age of 25 and over the age of 65 who were serving a sentence of less than one year’s imprisonment. Under the rules, this order is
a) Directly discriminatory
b) Indirectly discriminatory
c) Not discriminatory
d) Discriminatory, but justifiable.

167. Assume further that the government made a third order, releasing all graduate prisoners who are serving a sentence of less than one year’s imprisonment. Which of the following statistics would have to be true for this order to be indirectly discriminatory?
a) Only 13% of the prison population in Bihar have a graduation degree
b) Of the graduate prisoners 89% belong to upper castes
c) Only 25% women in Bihar get a graduation degree
d) All of the above.

Rule E: ‘A discriminatory act shall be justified if its effect is to promote the well-being of disadvantaged groups, such as women, dalits, religious minorities, sexual minorities or disabled persons.

168. Would the first Order of release of all women prisoners be justified under Rule E?
a) Yes because it promotes the well¬being, of women
b) No, because it does not promote the well-being of women prisoners
c) No, because it does not promote the well-being of all disadvantaged groups equally
d) None of the above.

Problem 4 (For questions 169 to 173)

Rules:
A. A minor is a person who is below the age of eighteen. However, where a guardian administers the minor’s property the age of majority is twenty-one.
B. A minor is not permitted by law to enter into a contract. Hence, where a minor enters into a contract with a major person, the contract is not enforceable. This effectively means that neither the minor nor the other party can make any claim on the basis of the contract.
C. In a contract with a minor, if the other party hands over any money or confers any other benefit on the minor, the same shall not be recoverable from the minor unless the other party was deceived by the minor to hand over money or any other benefit. The other party will have to show that the minor misrepresented her age, he was ignorant about the age of the minor and that he handed over the benefit on the basis of such representation.
Facts
Ajay convinces Bandita, a girl aged 18 that she should sell her land to him. Bandita’s mother Chaaru is her guardian. Nonetheless Bandita, without the permission of Chaaru, sells the land to Ajay for a total sum of rupees fifty lakh, paid in full and final settlement of the price. Chaaru challenges this transaction claiming that Bandita is a minor and hence the possession of the land shall not be given to Ajay. Thus Ajay is in a difficult situation and has no idea how to recover his money from Bandita.

169. Chaaru is justified in challenging the sale transaction because
a) Bandita is of unsound mind and is not in a position to make rational decisions
b) Though Bandita is eighteen years old, she will be treated as a minor, as Chaaru is her guardian
c) Though Bandita is eighteen year old, she cannot sell the land without the permission of her mother
d) Though Bandita is eighteen year old she should not be treated like a person who has attained the age of majority.

170. Ajay can be allowed to recover the money only if he can show that
a) He was deceived by Bandita who misrepresented her age
b) He honestly believed that Bandita was empowered under the law to sell the land
c) He was an honest person who had paid the full price, of the land to Bandita
d) Both (a) and (b).

171. In order to defend the sale, Bandita will need to show that
a) Bandita has attained the age of majority
b) Bandita is mature enough to make rational decisions regarding her own affairs
c) The sale transaction was beneficial to her interest and will enhance her financial status
d) None of the above.

172. Which of the following is correct?
a) Ajay should be allowed to recover the money because even though there is no contract, Bandita and Chaaru should not be allowed to unjustly benefit from Ajay’s money
b) Ajay should be allowed the possession of the land because Chaaru can always decide to approve the transaction between Ajay and Bandita
c) Ajay should not be allowed to recover because he induced Bandita, a minor, to sell the land
d) None of the above.

173. Which of the following is correct?
a) If Ajay is allowed to recover the money, that will defeat the law framed for protecting the minors against fraudulent persons
b) If Ajay is not allowed to recover that will cause him injustice as he has not paid the entire sale price
c) If Ajay is not allowed to recover, Chaaru will benefit from both the money and the land
d) None of the above.

Problem 5 (For questions 174 to 177)

Rules
A. The act of using threats to force another person to enter into a contract is called coercion.
B. The act of using influence on another and taking undue advantage of that person is called undue influence.
C. C In order to prove coercion, the existence of the use of threat, in any form and manner, is necessary. If coercion is proved, the person who has been so threatened can refuse to abide by the contract.
D. In order to prove undue-influence, there has to be a pre-existing relationship between the parties to a contract. The relationship has to be of such a nature that one is in a position to influence the other. If it is proven that there has been undue influence, the party who has been so influenced need not enforce the contract or perform his obligations under the contract.
Facts
Aadil and Baalu are best friends. Aadil is the son of multi-millionaire business person, Chulbul who owns Maakhan Pharmaceuticals. Baalu is the son of a bank employee, Dhanraj. One day, Aadil is abducted from his office by Baalu. Chulbul receives a phone call from Dhanraj telling him that if he does not make Baalu the CEO of Maakhan Pharmaceuticals, Aadil will be killed. Chulbul reluctantly agrees to make Baalu the CEO. Subsequently Chulbul and Baalu sign an employment contract. However as soon as Aadil is released and safely returns home, Chulbul tells Baalu that he shall not enforce the employment contract. Baalu and Dhanraj are not sure as to what is to be done next.

174. As per the rules and the given facts, who coerces whom?
a) Aadil coerces Baalu
b) Baalu coerces Chulbul
c) Dhanraj coerces Chulbul
d) None of the above.

175. In the above fact situation
a) There is undue influence exercised by Dhanraj on Baalu
b) There is undue influence exercised by Aadil on Chulbul
c) There is no undue influence
d) None of the above.

176. Chulbul is
a) Justified in refusing to enforce the employment contract as Chulbul was coerced by Dhanraj
b) Justified in refusing to enforce the employment contract as Baalu was complicity in the coercive act
c) Not justified in refusing to enforce the employment contract as Baalu was an innocent person and has not coerced Chulbul
d) Both (a) and (b).

177. Baalu will succeed in getting the employment contract enforced if he can show that
a) He is the best friend of Aadil
b) It was his father, and not he, who used coercion against Chulbul
c) Chulbul has promised his father to employ him
d) None of the above.

Problem 6 (For questions 178 to 181)

Rule A: When a State undertakes any measure, the effects of the measure must be the same for all those who are affected by it.
Facts
Hundred mountaineers embarked on an extremely risky climbing expedition in Leh. Weather conditions worsened five days into the expedition and the mountaineers are trapped under heavy snow. The government received information of this tragedy only two weeks after the unfortunate incident and has only 24-hours in which to send rescue helicopters. Weather stations across the world confirm that this particular region of Leh will experience blizzards of unprecedented intensity for almost two weeks after this 24 hour window rendering any helicopter activity in the region impossible and certain death for anyone left behind. The government has only five rescue helicopters with a maximum capacity of 50 people (excluding pilots and requisite soldiers) and these helicopters can fly only once in 24 hours to such altitudes.
As the Air Force gets ready to send the helicopters, an emergency hearing is convened in the Supreme Court to challenge this measure as this would leave 50 people to die.

178. If you were the judge required to apply Rule A, you would decide that
a) As many lives must be saved as possible
b) If everyone cannot be rescued, then everyone must be left behind
c) A measure cannot be upheld at the cost of 50 lives
d) It must be left to those who are trapped to decide if they want half amongst them to be saved and leave the rest to die.

Rule B: When a State undertakes any measure, everyone affected must have an equal chance to benefit from it.

179. As the government prepares to send in rescue helicopters, which option would be acceptable only under Rule B and not Rule A?
a) A lottery to choose the 50 survivors excluding those diagnosed with terminal illnesses from participating in the lottery
b) A lottery to decide the 50 survivors with single parents of children-below five years of age automatically qualifying to be rescued
c) The 50 youngest people should be rescued
d) None of the above.

180. Choosing 50 survivors exclusively by a lottery would be
a) Permissible under Rules A and B
b) Impermissible under Rules A and B
c) Permissible only under Rule B
d) Permissible only under Rule A.

181. If the government decides that it will either save everyone or save none, it would be
a) Permissible under Rules A and B
b) Impermissible under Rules A and B
c) Permissible only under Rule A
d) Permissible only under Rule B.

Problem 7 (For questions 182 to 186)

Rules
A. A person is an employee of another if the mode and the manner in which he or she carries out his work is subject to control and supervision of the latter.
B. An employer is required to provide compensation to his or her employees for any injury caused by an accident arising in the course of employment. The words “in the course of the employment’ mean in the course of the work which the employee is contracted to do and which is incidental to it.
Facts
Messrs Zafar Abidi and Co. (Company) manufactures bidis with the help of persons known as ‘patiadars’. The pattadars are supplied tobacco and leaves by the Company and are required to roll them into bidis and bring the bidis back to the Company. The pattadars are free to roll the bidis either in the factory or anywhere else they prefer. They are not bound to attend the factory for any fixed hours of work or for any fixed number of days. Neither are they required to roll up any fixed number of bidis. The Company verifies whether the bidis adhere to the specified instructions or not and pays the pattadars on the basis of the number of bidis that are found to be of right quality. Aashish Mathew is one of the pattadars of the Company. He was hit by a car just outside the precinct of the factory while he was heading to have lunch in a nearby food-stall. Aashish Mathew has applied for compensation from the Company.

182. Which of the following statements can most plausibly be inferred from the application of the rules to the given facts?
a) Aashish Mathew is an employee of the Company because the latter exercises control over the manner in which Aashish Mathew carries out his work
b) Aashish Mathew is not an employee but an independent contractor as he does not have a fixed salary
c) Aashish Mathew is an employee because the Company exercises control over the final quality of the bidis
d) Verification of the quality of bidis amounts to control over the product and not control over the mode and method of work and therefore, Aashish Mathew is not an employee of the Company.

183. In case the pattadars were compulsorily required to work in the factory for a minimum number of hours every day, then it would be correct to state that
a) The injury was not caused by an accident in the course of employment
b) Aashish Mathew would not be an employee as the Company would have still not exercised control over the manner of work
c) The injury suffered by Aashish Mathew could not be held to be one caused by an accident
d) Stipulations on place and hours of work relate to manner and mode of work and therefore, Aashish Mathew would be held to be an employee of the Company.

184. According to the facts and the rules specified, which of the following propositions is correct?
a) The Company is not liable to pay compensation as the injury to Aashish Mathew was not caused by an accident arising in the course of employment
b) The Company is liable to pay the compensation
c) Since the injury did not arise in the course of employment, the Company would not be liable to pay the compensation even though Aashish Mathew is an employee of the company
d) The Company is liable to pay the compensation as Aashish Mathew is a contracted pattadar with the company.

185.
Select the statement that could be said to be most direct inference from specified Facts
a) The injury to Aashish Mathew did not arise in the course of employment as he was not rolling bidis at the time when he was hit by the car
b) Since Ashish Mathew is a contracted pattadar with the Company, it shall be presumed that the injury was caused by an accident in the course of employment
c) Since there was no relationship of employment between Aashish Mathew and the Company, the injury suffered by Aashish Mathew could not be held to be one arising in the course of employment notwith¬standing the fact that the concerned injury was caused while he was involved in an activity incidental to his duties
d) As the concerned injury was caused to Aashish Mathew while he was involved in an activity incidental to his duties, the injury did arise in the course of employment.

186. If the pattadars were compulsorily required to work in the factory for a minimum number of hours every day, then the Company would have been liable to pay compensation to Aashish Mathew if the latter
a) Had been assaulted and grievously hurt by his neighbour inside the factory precincts over a property dispute
b) Had slipped and fractured his arm while trying to commute on a city bus from his home to the factory
c) Had been injured while commuting on a bus provided by the Company and which he was required by his contract to use every day
d) Had been caught in the middle of a cross-fire between police and a gang of robbers while travelling to work on a city bus.

Problem 8 (For questions 187 to 191)

Rules:
A. Whoever intending to take any moveable property out of the possession of any person without that person’s consent, moves that property out of his or her possession, is said to commit theft.
B. A person who, without lawful excuse, damages any property belonging to another intending to damage any such property shall be guilty of causing criminal damage.
C. Damage means any impairment of the value of a property.
Facts:
Veena, an old lady of 78 years, used to live with her grand-daughter Indira. Veena was ill and therefore bed-ridden for several months. In those months, she could not tolerate any noise and it became quite difficult to clean her room. After she died, Indira hired a cleaner, Lucky, to clean the room and throw away any rubbish that may be there.
There was a pile of old newspapers which Veena had stacked in a comer of her room. Lucky asked Indira if he should clear away the pile of old newspapers, to which she said yes. Lucky took the pile to a municipality rubbish dump. While Lucky was sorting and throwing away the newspapers, he was very surprised to find a beautiful painting in between two sheets of paper. He thought that Indira probably wouldn’t want this old painting back, especially because it was tom in several places and the colour was fading. He took the painting home, mounted it on a wooden frame and hung it on the wall of his bedroom. Unknown to him, the painting was an old masterpiece, and worth twenty thousand rupees. Before mounting the painting, Lucky pasted it on a plain sheet of paper so that it does not tear any more. By doing so, he made its professional restoration very difficult and thereby reduced its value by half.
Lucky’s neighbour Kamala discovered that the painting belonged to Indira. With the motive of returning the painting to Indira, Kamala climbed through an open window into Lucky’s room when he was away one afternoon and removed the painting from his house.

187. Has Lucky committed theft?
a) Yes, Lucky has committed theft of the newspapers and the painting
b) No, Lucky has not committed theft because he had Veena’s consent
c) Yes, Lucky has committed theft of the painting, but not of the newspapers
d) No, Lucky has not committed theft because he has not moved the painting out of Veena’s possession.

188. Is Lucky guilty of criminal damage?
a) No, Lucky is not guilty of criminal damage as he did not intentionally impair the value of the painting
b) Yes, Lucky is guilty of criminal damage as he intentionally stuck the paper on to the painting
c) No, Lucky is not guilty of criminal damage as he does not have the painting in his possession anymore
d) No, Lucky is not guilty of the criminal damage as he has not destroyed the painting.

189. If Lucky had discovered the painting before leaving Indira’s house rather than at the rubbish dump, would he have been guilty of theft in this case’?
a) Yes, he would be guilty of theft of the newspapers and the paintings
b) No, he would not be guilty of theft
c) Yes, he would be guilty of theft of the painting
d) None of the above.

190. Is Kamala guilty of theft’?
a) No, Kamala is not guilty of theft since the person she took the painting from (Lucky) was not its lawful owner
b) No, Kamala is not guilty of theft since she took the painting only with the motive of returning it to Indira
c) Yes, Kamala is guilty of theft as she took the painting out of Lucky’s possession without his consent
d) None of the above.

191. Which of the following propositions could be inferred from the facts and the rules specified?
a) Kamala is guilty of criminal damage as the person she took the painting from (Lucky) was not its lawful owner
b) Kamala is guilty of criminal damage as she took the painting without Lucky’s consent
c) Kamala is not guilty of criminal damage as the painting has not been completely destroyed
d) None of the above.

Problem 9 (For questions 192-195)

Rules:
A. When land is sold, all ‘fixtures’ on the land are also deemed to have been sold.
B. If a movable thing is attached to the land or any building on the land, then it becomes a ‘fixture’.
Facts:
Khaleeda wants to sell a plot of land she owns in Baghmara, Meghalaya and the sale
value decided for the plot includes the fully- furnished palatial six-bedroom house that she has built on it five years ago. She sells it to Gurpreet for sixty lakh rupees. After completing the sale, she removes the expensive Iranian carpet which used to cover the entire wooden floor of one of the bedrooms. The room had very little light and Khaleeda used this light- coloured radiant carpet to negate some of the darkness in the room. Gurpreet, after moving in, realises this and files a case to recover the carpet from Khaleeda.

192. As a judge you would decide in favour of
a) Gurpreet because when the price was agreed upon, Khaleeda did not inform her about removing the carpet
b) Gurpreet because the carpet was integral to the floor of the bedroom and therefore attached to the building that was sold
c) Khaleeda because a fully-furnished house does not entail the buyer to everything in the house
d) Khaleeda because by Virtue of being a carpet it was never permanently fixed to the floor of the building.

Assume that in the above fact scenario, Khaleeda no longer wants the carpet. She removes the elaborately carved door to the house after the sale has been concluded and claims that Gurpreet has no claim to the door. The door in question was part of Khaleeda’s ancestral home in Nagercoil, Tamil Nadu for more than 150 years before she had it fitted as the entrance to her Baghmara house.

193. As a judge you would decide in favour of
a) Khaleeda because while the rest of the building belongs to Khaleeda exclusively, the door is ancestral property and therefore the decision to sell it cannot be Khaleeda’s alone.
b) Gurpreet because the door is an integral part of the building as it is attached to it
c) Khaleeda because the door can be removed from the building and is therefore not attached to it.
d) Gurpreet because the contract is explicitly for the whole house and since the door is part of house, it cannot be removed subsequent to the sale.

194. Amongst the following options, the most relevant consideration while deciding a case on the basis of the above two principles would be
a) Whether the movable thing was included in the sale agreement
b) Whether the movable thing was merely placed on the land or building
c) Whether the movable thing had become an inseparable part of the land or building
d) Whether the movable thing could be removed.

195. Rule C: If a movable thing is placed on land with the intention that it should become an integral part of the land or any structure on the land it becomes a fixture.
Applying, Rules A and C, to the fact situations in questions 192 and 193, as a judge you would decide in favour of
a) Khaleeda in both situations
b) Gurpreet only in 192
c) Khaleeda only in 193
d) Gurpeet in both situations.

Problem 10 (For questions 196-200)

Rule A: An owner of land has the right to use the land in any manner he or she desires. The owner of land also owns the space above and the depths below it.
Rule B: Rights above the land extend only to the point they are essential to any use or enjoyment of land.
Rule C: An owner cannot claim infringement of her property right if the space above his or her land is put to reasonable use by someone else at a height at which the owner would have no reasonable use of it and it does not affect the reasonable enjoyment of his or her land.
Ramesh’s case: Ramesh owns an acre of land on the outskirts of Sullurpeta, Andhra Pradesh. The Government of India launches its satellites into space frequently from Sriharikota, near Sullurpeta. The Government of India does not deny that once the satellite launch has travelled the distance of almost 7000 kilometres it passes over Ramesh’s property. Ramesh files a case claiming that the Government of India has violated his property rights by routing its satellite over his property, albeit 7000 kilometre directly above it.

196. Applying only Rule A to Ramesh’s case, as a judge you would decide
a) In favour of the Government of India because the transgression was at a height at which Ramesh could not possibly have any use for
b) That ownership of land does not mean that the owner’s right extends infinitely into space above the land
c) In favour of Ramesh because he has the right to infinite space above the land he owns
d) In favour of the Government of India because it would lead to the absurd result that Ramesh and most other property owners would have a claim against airline companies and other countries of the world whose satellites orbit the earth.

Shazia’s case: Shazia owns a single storeyed house in Ahmedabad which has been in her family for more than 75 years. The foundation of the house cannot support another floor and Shazia has no intention of demolishing her family home to construct a bigger building. Javed and Sandeep are business partners and own three-storey houses on either side of Shazia’s house. Javed and Sandeep are also Ahmedabad’s main distributors for a major soft drink company. They have erected a huge hoarding advertising their products, with the ends supported on their roofs but the hoarding also passes over Shazia’s house at 70 feet and casts a permanent shadow on her terrace. Shazia decides to hoist a huge Indian flag, going up to 75 feet, on her roof. She files a case, asking the court to order Javed and Sandeep to remove the hoarding for all these reasons.

197. Applying only Rule B to Shazia’s case, you would decide in favour of
a) Javed and Sandeep because Shazia can easily hoist a flag below 70 feet
b) Shazia because she has the right to put her land to any use and the court cannot go into her intentions for hoisting a flag at 75 feet
c) Shazia because she has the absolute right to the space above her land
d) Javed and Sandeep because hoisting a flag 75 feet above one’s roof is not essential to the use and enjoyment of the land.

198. Applying only Rules A and B to Shazia’s case, you would decide
a) In favour of Shazia only under Rule A
b) In favour of Shazia under Rule A as well as B
c) Against Shazia under Rule B
d) Against Shazia under Rule A as well as B.

199. Applying only Rule B and C to Ramesh’s case, you would decide
a) In favour of Ramesh only under Rule B
b) In favour of Ramesh under Rule B as well as C
c) Against Ramesh under Rule C
d) Against Ramesh under Rule B as well as C.

200. Applying Rule C to Shazia’s case, you would decide
a) In her favour because hoisting a 75 feet high flag is reasonable
b) Against her because hoisting a 75 feet high flag is not reasonable
c) Against her because the hoarding is a reasonable use of the space above her land
d) In her favour because the permanent shadow cast by the hoarding affects the reasonable enjoyment of her land.

Download CLAT Past Year Papers with Full Solutions

We have compiled the CLAT Papers (2008 onwards) into one neat, beautifully formatted bundle for you to download, view offline or print. You can download it by clicking below

Download CLAT Question Papers & Solutions